Combinatorial proof that the number of even cardinality subsets is equal to the number of odd cardinality...












0












$begingroup$



Given a set of cardinality $ngeq 1$, the number of subsets of even cardinality is equal to the number of subsets of odd cardinality




I am looking for a combinatorial proof of this statement- I know an algebraic proof is easy, for instance by expanding $(1-1)^n$. If $n$ is odd it is easy to see there is a one-to-one correspondence between even-cardinality subsets an odd-cardinality subsets using the complement. However, I can't think of a combinatorial proof if $n$ is even. Any ideas?










share|cite|improve this question









$endgroup$

















    0












    $begingroup$



    Given a set of cardinality $ngeq 1$, the number of subsets of even cardinality is equal to the number of subsets of odd cardinality




    I am looking for a combinatorial proof of this statement- I know an algebraic proof is easy, for instance by expanding $(1-1)^n$. If $n$ is odd it is easy to see there is a one-to-one correspondence between even-cardinality subsets an odd-cardinality subsets using the complement. However, I can't think of a combinatorial proof if $n$ is even. Any ideas?










    share|cite|improve this question









    $endgroup$















      0












      0








      0





      $begingroup$



      Given a set of cardinality $ngeq 1$, the number of subsets of even cardinality is equal to the number of subsets of odd cardinality




      I am looking for a combinatorial proof of this statement- I know an algebraic proof is easy, for instance by expanding $(1-1)^n$. If $n$ is odd it is easy to see there is a one-to-one correspondence between even-cardinality subsets an odd-cardinality subsets using the complement. However, I can't think of a combinatorial proof if $n$ is even. Any ideas?










      share|cite|improve this question









      $endgroup$





      Given a set of cardinality $ngeq 1$, the number of subsets of even cardinality is equal to the number of subsets of odd cardinality




      I am looking for a combinatorial proof of this statement- I know an algebraic proof is easy, for instance by expanding $(1-1)^n$. If $n$ is odd it is easy to see there is a one-to-one correspondence between even-cardinality subsets an odd-cardinality subsets using the complement. However, I can't think of a combinatorial proof if $n$ is even. Any ideas?







      combinatorial-proofs






      share|cite|improve this question













      share|cite|improve this question











      share|cite|improve this question




      share|cite|improve this question










      asked Dec 19 '18 at 6:39









      Darren OngDarren Ong

      30219




      30219






















          2 Answers
          2






          active

          oldest

          votes


















          1












          $begingroup$

          Just pick your favourite element $a$. Now take a subset $X$ and add $a$ if $anotin X$
          and delete it if $ain X$. One gets a pairing of the subsets, and in each pair
          one subset is even, the other odd.






          share|cite|improve this answer









          $endgroup$





















            0












            $begingroup$

            use the case for odd $n$, and consider adding a single additional element to the set. the old subsets are still subsets, together with new ones which contain the additional element.






            share|cite|improve this answer









            $endgroup$














              Your Answer





              StackExchange.ifUsing("editor", function () {
              return StackExchange.using("mathjaxEditing", function () {
              StackExchange.MarkdownEditor.creationCallbacks.add(function (editor, postfix) {
              StackExchange.mathjaxEditing.prepareWmdForMathJax(editor, postfix, [["$", "$"], ["\\(","\\)"]]);
              });
              });
              }, "mathjax-editing");

              StackExchange.ready(function() {
              var channelOptions = {
              tags: "".split(" "),
              id: "69"
              };
              initTagRenderer("".split(" "), "".split(" "), channelOptions);

              StackExchange.using("externalEditor", function() {
              // Have to fire editor after snippets, if snippets enabled
              if (StackExchange.settings.snippets.snippetsEnabled) {
              StackExchange.using("snippets", function() {
              createEditor();
              });
              }
              else {
              createEditor();
              }
              });

              function createEditor() {
              StackExchange.prepareEditor({
              heartbeatType: 'answer',
              autoActivateHeartbeat: false,
              convertImagesToLinks: true,
              noModals: true,
              showLowRepImageUploadWarning: true,
              reputationToPostImages: 10,
              bindNavPrevention: true,
              postfix: "",
              imageUploader: {
              brandingHtml: "Powered by u003ca class="icon-imgur-white" href="https://imgur.com/"u003eu003c/au003e",
              contentPolicyHtml: "User contributions licensed under u003ca href="https://creativecommons.org/licenses/by-sa/3.0/"u003ecc by-sa 3.0 with attribution requiredu003c/au003e u003ca href="https://stackoverflow.com/legal/content-policy"u003e(content policy)u003c/au003e",
              allowUrls: true
              },
              noCode: true, onDemand: true,
              discardSelector: ".discard-answer"
              ,immediatelyShowMarkdownHelp:true
              });


              }
              });














              draft saved

              draft discarded


















              StackExchange.ready(
              function () {
              StackExchange.openid.initPostLogin('.new-post-login', 'https%3a%2f%2fmath.stackexchange.com%2fquestions%2f3046084%2fcombinatorial-proof-that-the-number-of-even-cardinality-subsets-is-equal-to-the%23new-answer', 'question_page');
              }
              );

              Post as a guest















              Required, but never shown

























              2 Answers
              2






              active

              oldest

              votes








              2 Answers
              2






              active

              oldest

              votes









              active

              oldest

              votes






              active

              oldest

              votes









              1












              $begingroup$

              Just pick your favourite element $a$. Now take a subset $X$ and add $a$ if $anotin X$
              and delete it if $ain X$. One gets a pairing of the subsets, and in each pair
              one subset is even, the other odd.






              share|cite|improve this answer









              $endgroup$


















                1












                $begingroup$

                Just pick your favourite element $a$. Now take a subset $X$ and add $a$ if $anotin X$
                and delete it if $ain X$. One gets a pairing of the subsets, and in each pair
                one subset is even, the other odd.






                share|cite|improve this answer









                $endgroup$
















                  1












                  1








                  1





                  $begingroup$

                  Just pick your favourite element $a$. Now take a subset $X$ and add $a$ if $anotin X$
                  and delete it if $ain X$. One gets a pairing of the subsets, and in each pair
                  one subset is even, the other odd.






                  share|cite|improve this answer









                  $endgroup$



                  Just pick your favourite element $a$. Now take a subset $X$ and add $a$ if $anotin X$
                  and delete it if $ain X$. One gets a pairing of the subsets, and in each pair
                  one subset is even, the other odd.







                  share|cite|improve this answer












                  share|cite|improve this answer



                  share|cite|improve this answer










                  answered Dec 19 '18 at 6:49









                  Lord Shark the UnknownLord Shark the Unknown

                  107k1162135




                  107k1162135























                      0












                      $begingroup$

                      use the case for odd $n$, and consider adding a single additional element to the set. the old subsets are still subsets, together with new ones which contain the additional element.






                      share|cite|improve this answer









                      $endgroup$


















                        0












                        $begingroup$

                        use the case for odd $n$, and consider adding a single additional element to the set. the old subsets are still subsets, together with new ones which contain the additional element.






                        share|cite|improve this answer









                        $endgroup$
















                          0












                          0








                          0





                          $begingroup$

                          use the case for odd $n$, and consider adding a single additional element to the set. the old subsets are still subsets, together with new ones which contain the additional element.






                          share|cite|improve this answer









                          $endgroup$



                          use the case for odd $n$, and consider adding a single additional element to the set. the old subsets are still subsets, together with new ones which contain the additional element.







                          share|cite|improve this answer












                          share|cite|improve this answer



                          share|cite|improve this answer










                          answered Dec 19 '18 at 6:48









                          David HoldenDavid Holden

                          14.9k21225




                          14.9k21225






























                              draft saved

                              draft discarded




















































                              Thanks for contributing an answer to Mathematics Stack Exchange!


                              • Please be sure to answer the question. Provide details and share your research!

                              But avoid



                              • Asking for help, clarification, or responding to other answers.

                              • Making statements based on opinion; back them up with references or personal experience.


                              Use MathJax to format equations. MathJax reference.


                              To learn more, see our tips on writing great answers.




                              draft saved


                              draft discarded














                              StackExchange.ready(
                              function () {
                              StackExchange.openid.initPostLogin('.new-post-login', 'https%3a%2f%2fmath.stackexchange.com%2fquestions%2f3046084%2fcombinatorial-proof-that-the-number-of-even-cardinality-subsets-is-equal-to-the%23new-answer', 'question_page');
                              }
                              );

                              Post as a guest















                              Required, but never shown





















































                              Required, but never shown














                              Required, but never shown












                              Required, but never shown







                              Required, but never shown

































                              Required, but never shown














                              Required, but never shown












                              Required, but never shown







                              Required, but never shown







                              Popular posts from this blog

                              Plaza Victoria

                              In PowerPoint, is there a keyboard shortcut for bulleted / numbered list?

                              How to put 3 figures in Latex with 2 figures side by side and 1 below these side by side images but in...